Proving h is a Differential Form

Click For Summary
The discussion focuses on proving that the expression h = e_1∧e_2 + e_3∧e_4 is a differential form, where e_1, e_2, e_3, and e_4 represent basis elements. It is clarified that these basis elements can be interpreted as differentials, such as dx, dy, dz, and dt, leading to the expression being equivalent to dxdy + dzdt. The anti-symmetry property of the wedge product is highlighted, noting that terms like dx∧dx equal zero. The inquiry remains about demonstrating the smoothness of h as a differential form, emphasizing the need to show it is a smooth section of the projection map. The conversation underscores the importance of understanding the definitions and properties of differential forms in this context.
Canavar
Messages
15
Reaction score
0
Hello,

I try to understand differential forms. For istance i want to prove that
h=e_1\wedge e_2 + e_3\wedge e_4
is a differential form, where e_1,..,e_4 are elements of my basis.



Do you have a idea, why this is a differential form?

Regards
 
Physics news on Phys.org
First, your 'basis' elements are themselves "differentials". If you think of e_1= dx, e_2= dy, e_3= dz, and e_4= dt then e_1\wedge e_2+ e_3\wedge e_4= dxdy+ dzdt. It would convert the function f(x,y,z, t) into
\int\int f(x,y,z,t) dxdy+ f(x,y,z,t)dzdt

The "wedge", \wedge, is there because this product is "anti-symmetric" dx\wedge dy= -dy\wedge dx so the, in particular, such things as "dx\wedge dx" will be 0.
 
Hello,

thank you, but why it is a differential form? We have defined differential form as a smooth section of the projection map.
Therefore i have to show this. But for instance i do not see why it is smooth.

Regards
 
Question: A clock's minute hand has length 4 and its hour hand has length 3. What is the distance between the tips at the moment when it is increasing most rapidly?(Putnam Exam Question) Answer: Making assumption that both the hands moves at constant angular velocities, the answer is ## \sqrt{7} .## But don't you think this assumption is somewhat doubtful and wrong?

Similar threads

  • · Replies 14 ·
Replies
14
Views
3K
  • · Replies 2 ·
Replies
2
Views
2K
  • · Replies 18 ·
Replies
18
Views
6K
  • · Replies 4 ·
Replies
4
Views
2K
  • · Replies 4 ·
Replies
4
Views
2K
  • · Replies 13 ·
Replies
13
Views
2K
  • · Replies 6 ·
Replies
6
Views
3K
  • · Replies 7 ·
Replies
7
Views
2K
  • · Replies 2 ·
Replies
2
Views
1K